How does one show that the function is differentiable?

Click For Summary
To demonstrate that the function (x^2+y^2)^\alpha is differentiable at (0,0) for α > 1/2, it's essential to show the continuity of its partial derivatives at that point. The expression 2xα(x^2+y^2)^{α-1} needs to be evaluated for continuity at (0,0), which can be complex. A suggested approach involves breaking the problem into two cases based on the value of α and applying the squeeze theorem for one of those cases. Utilizing the limit definition of the partial derivative can also clarify the continuity argument. Overall, focusing on these strategies can effectively demonstrate differentiability.
Feynman's fan
Messages
14
Reaction score
0
I'd like to show that if \alpha>\frac{1}{2} then (x^2+y^2)^\alpha is differentiable at (0,0).

The usual way is to show that the partial derivatives are continuous at (0,0).

Yet I am a little confused how to show that 2x\alpha(x^2+y^2)^{\alpha-1} is continuous at (0,0). I have tried working it out by definition, yet it seems to be a mess.

Any hints are very appreciated!
 
Physics news on Phys.org
Feynman's fan said:
I'd like to show that if \alpha>\frac{1}{2} then (x^2+y^2)^\alpha is differentiable at (0,0).

The usual way is to show that the partial derivatives are continuous at (0,0).

Yet I am a little confused how to show that 2x\alpha(x^2+y^2)^{\alpha-1} is continuous at (0,0). I have tried working it out by definition, yet it seems to be a mess.

Any hints are very appreciated!

I might be wrong, so you should wait until others give advice, but I believe something that can get you started would be to first break this into two cases for the values of ##\alpha##. After that, I would use a squeeze theorem argument for one of the cases.
 
Simon Bridge,
thank you, it's all clear now!
 
No worries - happy New Year.
 
Question: A clock's minute hand has length 4 and its hour hand has length 3. What is the distance between the tips at the moment when it is increasing most rapidly?(Putnam Exam Question) Answer: Making assumption that both the hands moves at constant angular velocities, the answer is ## \sqrt{7} .## But don't you think this assumption is somewhat doubtful and wrong?

Similar threads

  • · Replies 2 ·
Replies
2
Views
1K
  • · Replies 14 ·
Replies
14
Views
2K
  • · Replies 4 ·
Replies
4
Views
2K
  • · Replies 10 ·
Replies
10
Views
2K
  • · Replies 9 ·
Replies
9
Views
1K
  • · Replies 11 ·
Replies
11
Views
2K
Replies
4
Views
2K
  • · Replies 18 ·
Replies
18
Views
3K
Replies
26
Views
2K
Replies
6
Views
2K